[PDF] [PDF] Practice Problems 3 : Cauchy criterion, Subsequence

(c) If (xn) satisfies the Cauchy criterion, then there exists an α ∈ R such that 0



Previous PDF Next PDF





[PDF] Week 3 Solutions Page 1 Exercise (241) Prove that ) is Cauchy

(n2−1 n2 ) is Cauchy using directly the definition of Cauchy sequences Proof Given ϵ > 0 Let {xn} be a sequence such that there exists a 0



[PDF] Midterm Solutions

Let X = (xn) be a sequence of positive real numbers such that lim (xn+1 xn ) is Cauchy 4 Let (fn) ∈ C[0,1] be such that there exists M > 0 such that fn ∞ ≤



[PDF] MATH 403 ANALYSIS I - SPRING 2010 SOLUTIONS to

A real sequence (xn) is called contractive if there exists a constant 0 0 and let N be such that x2 − x1 Cn−1 1 1−C



[PDF] Practice Problems 3 : Cauchy criterion, Subsequence

(c) If (xn) satisfies the Cauchy criterion, then there exists an α ∈ R such that 0



[PDF] Let xn be a monotone increasing sequence bounded above and con

every monotone increasing bounded sequence bounded above converges that for every ε > 0 there exists some n > 1/ε such that xn < ε 1 (c) Using the Archimedean property, argue that yk cannot be bounded above by M, hence



[PDF] MA 101 (Mathematics I) Hints/Solutions for Practice Problem Set - 2

If xn → 0, then there exists n0 ∈ N such that xn < 1 2 If possible, let there exist a non-convergent sequence Hence there exists c ∈ (0,∞) such that f (c) = 0



[PDF] Sequences - UC Davis Mathematics

(xn)∞ n=0 is a function f : N0 → R where xn = f(n) and N0 = {0, 1, 2, 3, }, and eventually; and (xn) does not converge to x ∈ R if there exists ϵ0 > 0 such that We let x = lim n→∞ xn, y = lim n→∞ yn The first statement is immediate if c = 0



[PDF] M17 MAT25-21 HOMEWORK 5 SOLUTIONS 1 To Hand In

(c) A divergent monotone sequence with a Cauchy subsequence Let (an) and (bn) be Cauchy sequences Decide whether each k=1 ak is Cauchy if and only if for all ϵ > 0 there exists N ∈ N such that whenever n>m ≥ N (b) A convergent series ∑xn and a bounded sequence (yn) such that ∑xnyn diverges (c) Two 



[PDF] MATH 409, Summer 2019, Practice Problem Set 2 - TAMU Math

29 mai 2019 · Show that a sequence (xn)∞ n=1 is convergent to l ∈ R, if and only if for every ε ∈ (0, 2) there exists N ∈ N such that for all n ⩾ N, xn − l < ε

[PDF] letra cancion bandolero paris latino

[PDF] letter and sound assessment form

[PDF] letter coding examples

[PDF] letter coding tricks

[PDF] letter granting permission to use copyrighted music

[PDF] letter identification assessment free

[PDF] letter identification assessment pdf

[PDF] letter of acceptance of appointment as director

[PDF] letter of appointment of additional director in private company

[PDF] letter of consent for child to travel with grandparents

[PDF] letter of consent to travel with one parent

[PDF] letter of permission to travel

[PDF] letter of permission to travel from employer

[PDF] letter pattern programs in java

[PDF] letter stating i pay rent

Practice Problems 3 : Cauchy criterion, Subsequence

1. Show that the sequence (xn) dened below satises the Cauchy criterion.

(a)x1= 1 andxn+1= 1 +1x nfor alln1 (b)x1= 1 andxn+1=12+x2nfor alln1: (c)x1= 1 andxn+1=16 (x2n+ 8) for alln1:

2. Let (xn) be a sequence of positive real numbers. Prove or disprove the following

statements. (a) Ifxn+1xn!0 then (xn) converges. (b) Ifjxn+2xn+1j0< <1 andjxn+1xnj jxnxn1jfor alln2N.

3. Let (xn) be a sequence of integers such thatxn+16=xnfor alln2N. Prove or

disprove the following statements. (a) The sequence (xn) does not satisfy the Cauchy criterion. (b) The sequence (xn) cannot have a convergent subsequence.

4. Suppose that 0< <1 and that (xn) is a sequence satisfying the condition:

jxn+1xnj n; n= 1;2;3;::::Show that (xn) satises the Cauchy criterion.

5. Let (xn) be dened by:x1=11!

;x2=11! 12! ;:::;xn=11! 12! +:::+(1)n+1n!:Show that the sequence converges.

6. Let 1x1x22 andxn+2=px

n+1xn,n2N. Show thatxn+1x n12 for all n2N,jxn+1xnj 23 jxnxn1jfor alln2Nand (xn) converges.

7.(*)Show that a sequence (xn) of real numbers has no convergent subsequence if

and only ifjxnj ! 1.

8.(*)Let (xn) be a sequence inRandx02R. Suppose that every subsequence of

(xn) has a convergent subsequence converging tox0. Show thatxn!x0.

9.(*)Let (xn) be a sequence inR. We say that a positive integernis a peak of the

sequence ifm > nimpliesxn> xm(i.e., ifxnis greater than every subsequent term in the sequence). (a) If (xn) has innitely many peaks, show that it has a decreasing subsequence. (b) If (xn) has only nitely many peaks, show that it has an increasing subsequence. (c) From (a) and (b) conclude that every sequence inRhas a monotone subse- quence. Further, every bounded sequence inRhas a convergent subsequence (An alternate proof of Bolzano-Weierstrass Theorem).

Hints/Solutions

1. (a) Note thatjxn+1xnj=j1x

n1x n1j=jxn1xnx nxn1jandjxnxn1j=j(1+1x n1)xn1j= jxn1+ 1j 2. This implies thatjxn+1xnj 12 jxnxn1j:Hence (xn) satises the contractive condition and therefore it satises the Cauchy criterion. (b) Observe thatjxn+1xnj=jx2nx2n1j(2+x2n)(2+x2n1)jxnxn1jjxn+xn1j4 24
jxnxn1j: (c) We havejxn+1xnj jxnxn1jjxn+xn1j6 46
jxnxn1j:

2. (a) False. Choosexn=pnand observe thatxn+1xn=1pn+1+pn

!0. (b) False. Forxn=pn,jxn+2xn+1j=jpn+ 2pn+ 1j<1pn+1+pn =jxn+1xnj. (c) False. Takexn=1n . Ifj1n+11n j j1n

1n1jfor some >0, show that1:

3. (a) True. Becausejxn+1xnj90 asn! 1.

(b) False. Considerxn= (1)n.

4. Letn > m. Thenjxnxmj jxnxn1j+jxn1xn2j++jxm+1xmj

n1+n2++m=m[1 +++n1+m]m1!0 as m! 1.

Thus (xn) satises the Cauchy criterion.

5. Use Problem 4.

6. Since 1xn2,xn+1x

n12 . Observe thatx2n+1x2n=xnxn1x2n=xn(xn1xn):

Thereforejxn+1xnj=jxnx

n+1+xnjjxn1xnj 23 jxnxn1j.

7. Supposejxnj ! 1. If (xnk) is a subsequence of (xn), then observe thatjxnkj ! 1.

Ifjxnj91, then there exists a bounded subsequence of (xn). Apply Bolzano-

Weierstrass theorem.

8. Supposexn9x0. Then there exists0>0 and a subsequence (xnk) of (xn) such

thatjxnkx0j 0for allnk. Note that (xnk) has no subsequence converging to x 0.

9. (a) If (xn) has innitely many peaks,n1< n2< ::: < nj< ::::Then the subsequence

(xnj) is decreasing. (b) Suppose there are only nite peaks and letNbe the last peak. Sincen1=N+1 is not a peak, there existsn2> n1such thatxn2xn1:Againn2> Nis not a peak, there existsn3> n2such thatxn3xn2:Continuing this process we nd an increasing sequence (xnk).quotesdbs_dbs6.pdfusesText_12